Blood Bank

¡Supera tus tareas y exámenes ahora con Quizwiz!

Approximately what percent of the overall population is Rh negative:

15 % Approximately 15% of individuals are Rh negative, while approximately 85% of individuals are Rh positive.

Platelets that are not collected by an apheresis method must be prepared within ________ of the collection of Whole Blood.

8 hours Platelets that are not collected by an apheresis method must be prepared within 8 hours of the collection of Whole Blood. The Whole Blood must not be cooled below 20oC to allow appropriate storage of Platelets at room temperature (20 - 24oC). Apheresis Platelets would be prepared according to the Apheresis instrument manufacturer"s instructions.

The accepted interval between blood donations is:

8 weeks The mandatory waiting period between blood donations is 56 days, or eight weeks.

Which of the following represents the approximate percentage of the population that is Rh positive:

85 %

If an average-weight adult male patient with a 7 gram/dL hemoglobin is given two units of packed cells, what would be the approximate new hemoglobin value (assuming there is no active bleeding or other predisposing factors that would shorten the survival of the blood cells

9.0 gm/dL One unit of Red Blood Cells increases the hemoglobin level by approximately 1g/dL in an adult who is not actively bleeding and has no other predisposing factors that would shorten the survival of the transfused blood cells.

The Kidd antibody is MOST commonly associated with:

Delayed hemolytic transfusion reactions

Which of the following antigen groups is closely related to the ABO antigens on the red cell membrane?

I, i

After the second spin in the preparation of platelets from whole blood, the platelet products should be:

Allowed to rest for 1-2 hours.

Which of the following blood components will provide the best source of fibrinogen for a patient with hypofibrinogenemia:

Cryoprecipitate Cryoprecipitate is the cold-insoluble protein remaining after FFP is thawed slowly at 1-6 degrees Celsius. It contains about 50% of the factor VIII, and 20-40% of the fibrinogen present in the original fresh plasma. It also contains some factor XIII, and Von Willebrand factor.

Which of the following blood components contain the most factor VIII concentration relative to the unit volume?

Cryoprecipitated AHF

A primary immune response is generally associated with which antibody?

IgM

The majority of anti-Lea antibodies are of which immunoglobulin class?

IgM

Match the following antibodies to their appropriate immunities:

IgM Anti-I IgG Anti-K IgG Anti-Jka IgM Anti-M IgM Antibodies: anti-M, anti-I IgG Antibodies: anti-K, anti-Jka

Rh immune globulin therapy in postpartum women provides:

Passive protection

Transfusion-associated graft versus host disease (TA-GVHD) may be prevented by which of the following?

Irradiation of cellular products

Which specific terminal sugar causes a red cell to have A antigenic activity?

N-acetylgalactosamine

An individual with type AB blood will demonstrate the complete absence of which of the following antigen sites?

None of the above The A and B antigens are present on the red cells of an AB patient. H antigen is a precursor to the ABO antigens.

What are the possible ABO genotypes of offspring from parents whose genotypes are OO and AB:

OA OB

Antibodies in the Rh system typically exhibit which one of the following characteristics?

Reacts best at 37ºC and AHG

A group Oh Bombay individual's blood specimen can be differentiated from blood specimens of normal group O persons by:

Serum containing anti-H

When given during pregnancy, RhIg may cross the placenta and sensitize fetal D-positive RBCs.

True Because RhIg contains IgG anti-D, it can cross the placenta and sensitize fetal Rh positive red cells. Affected infants may be born with a weakly positive DAT, but significant hemolysis does not occur.

Antihuman globulin (AHG) enables sensitized red cells to cross-link so that agglutination is visible.

True The mode of action of AHG is to join red cells that have the antigen antibody complex attached to the surface. When AHG is added, the anti human globulin attaches to the antigen-antibody complex and links with other cells that have this attachment. Visible agglutination occurs when they are joined.

All of the following criteria for donor RBC to be used for an exchange transfusion relate to both ABO HDFN and HDFN due to anti-D: Less than or equal to 7 days old (or fresher) Reconstituted with AB FFP CMV negative Negative for hemoglobin S Irradiated

True The statement is true. The listed criteria apply to donor RBC for exchange transfusions to treat any type of HDFN, including those caused by antibodies outside the ABO and Rh blood group systems.

A rosette test to screen for Fetal Maternal Hemorrhage should not be performed if the newborn has a weak D antigen.

True The statement is true. A rosette test cannot be done to screen for FMH if the fetus is weak D because false negatives may result.

An issued unit of blood may be returned to blood bank inventory if it has not been outside a monitored refrigerator for longer than:

30 minutes

How many out of 1,000,000 Caucasians will have the following phenotype: Group 0, K+, Jk(a+)

30,000 Group O blood types comprise about 44% of the caucasian population. Out of 1,000,000 people, this leaves us with about 440,000 people. Approximately 9% of caucasians have the K antigen, leaving us with 39,600 individuals who are group O, K+. Approximately 77% of caucasians are positive for the Jka antigen. 77% of 39,600 individuals leaves us with 30,492 or approximately 30,000 individuals that are group O, K+ and Jk(a+).

How many micrograms (ug) of IgG anti-D are in a 1500 IU vial of RhIg?

300 Feedback 300 ug of IgG anti-D are in a 1500 IU vial of RhIg. The conversion rate is one ug = 5 IU. Question Difficulty: Level 4

What is the MINIMUM number of days that samples from blood donor and recipient be kept following blood transfusion?

7

Donor and recipient blood samples must be kept for at least how long after transfusion?

7 days

If possible, Rh immune globulin should be administered within what period of time following the delivery of an Rh positive or weak-D positive infant, or other Rh alloimmunizing event?

72 hours

The A1 subgroup represents approximately what percentage of group A individuals:

75 - 80%

If the parents are group A and B respectively, what are the possible blood groups of their children?

A and B and AB and O If the parents of children are type A and B, then their possible allele combinations are type AA or AO and type BB or BO. If the parents carried the AO and BO alleles, then their children could have the following blood types if one allele from each parent was passed to the child: AB, AO, OB, OO = possibility of types AB, A, B or O

Your screen cells are 3+ at immediate spin and weak (W)+ at AHG. Your auto control is negative for both phases. Some of your antibody panel cells are 3+ at immediate spin and negative at AHG. What should you suspect?

A cold antibody may be present

Which of the following blood group system antibodies is the direct antiglobulin test (DAT) most UNRELIABLE in helping to diagnose hemolytic disease of the fetus and newborn (HDFN)?

ABO In ABO HDFN, the DAT may be weakly positive or negative, making it an unreliable test. The DAT is usually reliable for antibodies in the other listed blood group systems that cause HDFN.

Which of the following steps should be taken in the IMMEDIATE investigation of a potential hemolytic transfusion reaction? (choose all that apply)

ABO and DAT on the post-transfusion patient sample Check for a clerical error Visual examination of the post-reaction and pre-reaction (if available) plasma for hemolysis

Units of A-, B-, and O-negative red blood cells are shipped to your transfusion service. What testing MUST be performed by your facility prior to placing these units into your inventory?

ABO and Rh All donor Red Blood Cell units must have the ABO group confirmed. The D negativity of units that are labeled as "Rh-negative" must also be confirmed. Tests for weak D are not required.

The presence of hemolysis in a post-transfusion blood sample would probably indicate what?

ABO incompatibility

The most severe acute hemolytic transfusions reactions are the result of which of the following:

ABO incompatibility Incompatibility in blood groups other then ABO may also cause acute hemolysis, but the reactions are rarely as severe.

Autologous blood must be tested for which of the following before transfusion:

ABO, Rh

Which of the following tests must be repeated by the lab on homologous blood received from the Red Cross or other community blood sources:

ABO, Rh The Red Cross and the community blood centers have already performed serological testing for infectious agents.

At what phase of antibody screen testing is it most important to read the reactions in order to detect clinically significant antibodies?

AHG

A false-negative reaction while performing the DAT technique may be the result of:

AHG addition delayed for 40 or more minutes

A febrile nonhemolytic transfusion reaction is characterized by which of the following?

An increase in temperature of >1 degree C above 37 degrees during transfusion

Which of the following antibodies is the most common cause of severe hemolytic disease of the newborn:

Anti-D Feedback Anti-D is the antibody most commonly associated with severe hemolytic disease of the newborn, or HDN. Though ABO incompatibilities can also cause HDN, anti-D causes the highest incidence of severe HDN. Question Difficulty: Level 2

If an Rh negative patient is administered a unit of R1R1 packed red cells, which one of the following antibodies would be most likely to develop:

Anti-D R1R1 (DCe/DCe) cells are positive for the D antigen, which is the most immunogenic antigen of the Rh system, followed by c and E.

Which one of the following antibodies has been implicated in hemolytic disease of the fetus and newborn (HDFN)?

Anti-K Only IgG antibody can cross the placental barrier and enter the fetal circulation to attach to the specific antigen on the fetal red cells resulting in HDFN. IgM antibodies, which are larger than IgG, do not cross the placenta and therefore would not be implicated in HDFN. Anti-K is the only IgG antibody listed above. Both Anti-I and Anti Lea are IgM antibodies.

What is a transfusion reaction?

Any adverse event associated with the transfusion of blood components

Type B blood is found in higher frequency in:

Asians

The incidence of hemolytic disease of the fetus and newborn (HDFN) due to anti-D would be expected to be rarest in which population?

Asians Recognizing that considerable diversity exists in populations, of those listed, Asians have the highest frequency of being D positive and therefore are unable to produce anti-D.

Match each blood type with the corresponding antibody you would find in its serum:

B Anti-A A Anti-B O Anti-A, Anti-B AB none

An error was made in a handwritten blood bank record. Which of the following methods is acceptable for correcting the error to remain in compliance with current good manufacturing practices requirements?

B. Draw a single line through the error so that it can still be read.

At what frequency should quality control testing be performed on each lot of anti-human globulin to be in compliance with the FDA's current good manufacturing practices requirements?

B. Each day of use

What additional information is required on a label or tie tag of an autologous unit?

B. Identification of the recipient

Which of the following conditions is most frequently associated with anti-I:

Cold agglutinin diseaseCold agglutinin disease

Which is the best component to treat a patient with fibrinogen deficiency?

Cryoprecipitated AHF Crypoprecipitated AHF is the only product that contains a concentrated amount of fibrinogen.

Which of the following is used as a source for irradiation of blood products:

Cs137 Irradiation is performed using cesium-137 or cobalt-60 in self-contained blood irradiators or hospital radiation therapy machines.

If a patient has an antibody to a low-frequency antigen like Kpa or Jsa, how would this impact the ability to find compatible blood and to identify the antibody?

Easy to find compatible blood but difficult to identify the antibody

Patients with which of the following conditions would benefit most from washed red cells:

Elevated serum potassium

When performing a transfusion reaction investigation, what is the clerical check used to detect?

Errors that may cause an ABO-incompatibility due to patient or donor unit mix up

A severe hemophiliac, with a Factor VIII activity of less than 1%, is actively bleeding due to a serious accident. The blood product of choice is:

Factor VIII concentrate

IgG coated red cells are added to negative antiglobulin tests to detect which of the following sources of error:

Failure to add Coombs serum

Maternal antibody titer is a good indicator of severity of HDFN.

False Maternal antibody titer is an unreliable indicator of severity of HDFN and is used to determine if clinical fetal monitoring is warranted.

ABO incompatibility between a D-negative mother and a D-positive fetus eliminates the possibility of HDFN due to anti-D.

False ABO incompatibility between mother and fetus decreases the incidence of HDFN due to anti-D but does not eliminate it entirely

A pregnant female who received RhIg at 28 weeks gestation has a positive antibody screen at delivery. If the antibody has been confirmed as anti-D alone and reacts 1+ in the indirect antiglobulin test with D+ red cells, performing a titration to investigate if the anti-D is immune is good practice.

False Because lower titers could be due to both passive and immune anti-D, in the absence of results that suggest immune anti-D, routine antibody titration is not a good use of time compared to assuming that anti-D is passive. Best practice guidelines do NOT recommend routine titration for women known to be injected with RhIg and exhibiting a 2+ or less reaction with D+ red cells consistent with passive anti-D from RhIg.

Blood safety standards such as AABB Standards specify that a computer crossmatch is acceptable when an Rh negative woman has an anti-D consistent with antenatal RhIg administration.

False Blood safety standards do not address whether a computer crossmatch is acceptable when what appears to be RhIg-derived passive anti-D is present. Laboratories are left to develop their own policies.

A pregnant female has been injected with RhIg antenatally and has a positive antibody screen at delivery. If the antibody has been confirmed as anti-D alone and reacts only weakly (1+ in the indirect antiglobulin test), the anti-D is definitely passive.

False It is impossible to differentiate passive from immune anti-D when the antibody reacts only weakly. Given that the mother has been injected with RhIg, it is likely that the anti-D is passive, but not definite. A weak anti-D could be either. It's only if the anti-D is strong that passive anti-D can be discounted.

Reverse typing is done using known antisera to detect ABO antigens present on the patient's red blood cells.

False Reverse typing is performed using reagent red cells with known antigens to detect ABO antibodies present in the patient's serum.

When performing an antibody panel, which one of the following antigens will be rendered inactive by enzyme treatment?

Fya Duffy antigens are rendered inactive by enzyme treatments since the enzyme will remove the sialic acid from the RBC membrane. This process destroys: M, N, S, s, and Duffy antigens. The process enhances the reactions for Rh, Kidd, Lewis, I, and P antigens.

Which of the following blood group antigens are most susceptible to destruction by the action of enzymes:

Fya Duffy, or Fya and Fyb antigens, are most sensitive to enzyme treatment since they will be destroyed during this process. Enzyme panels can be helpful when multiple antibodies, including duffy, are present in a patient sample. With the duffy antigens destroyed, the panel can be performed to identify the remaining antibodies present.

A patient has a probable anti-Fya but anti-c and anti-K have not been excluded. Which of the following cells would be the one most useful cell to exclude both anti-c and anti-K efficiently in this patient?

Fya-; Fyb+; C-c+; K+k- The ideal red cell to exclude both antibodies efficiently is one that is Fy(a-) and homozygous positive for both c and K: Fy(a-b+)C-c+K+k- The other red cells exclude only anti-c or anti-K (not both) or are Fy(a+).

Antibodies to which of the following are the most frequent cause of febrile transfusion reactions:

Granulocytes

The parents' blood types were AB and O. The blood group of any of their children could ONLY be:

Group A or B only

DR antigens are found in which of the following systems:

HLA system Feedback HLA-DR is a MHC class II(major histocompatibility complex) surface receptor on the cellular surface which is encoded by the human leukocyte antigen complex, or HLA. Question Difficulty: Level 3

Which of the following antigens is classified as a Major Histocompatibility Complex Class II antigen (MHCII)?

HLA-DR HLA-DR is a class II MHC. HLA-A, HLA-B, and HLA-C are all class I MHC.

All of the following cellular antigens are important to an immunohematologist except:

Haptens A hapten is an incomplete antigen.

The chief purpose of performing a standard crossmatch is to :

Identify recipient antibodies against donor cells

Which of the following is most commonly associated with febrile non-hemolytic transfusion reactions:

Immune response to leukocytes Feedback The immune response in the recipient to donor leukocytes present in a blood product is most commonly associated with febrile, non-hemolytic transfusion reactions. Question Difficulty: Level 5

Which of the following statements best describes Rh antibodies:

Immune, IgG Rh antibodies are of the IgG class of immunoglobulins, and occur as a result of exposure to the corresponding antigens, through pregnancy or transfusion.

Acute intravascular hemolysis as the result of a blood transfusion is most often associated with which of the following causes?

Transfusion of ABO incompatible red cells Acute hemolytic transfusion reactions are most commonly due to ABO-incompatible blood being transfused to a recipient with naturally occurring ABO alloantibodies (anti-A, anti-B, anti A,B).

What is the MOST likely cause of the ABO discrepancy when the following results were obtained from a first-time 29-year old, blood donor? Forward Group Anti-A = Negative Anti-B= Negative Reverse Group A1 Cells = Negative B Cells = 3+

Weak subgroup of A It is important to recognize that a weak subgroup of A may be present if this blood is from a donor. If the blood were transfused as a group O to a group O recipient, a transfusion reaction would occur.

If Jka is showing dosage, how might reactions on an antibody panel appear?

Weaker if heterozygous for Jka and Jkb

Which one of the following is NOT a cause for donor deferral?

Weighs 115 pounds

The following steps must be followed in preparation of a platelet concentrate:

Whole blood centrifuged at low speed - plasma separated then centrifuged at high speed

Which organism is MOST likely responsible for septic reactions associated with Red Blood Cell transfusions?

Yersina entercolitica

Which one of the following tests BEST correlates with the severity of hemolytic disease of the newborn (HDN).

amniotic fluid bilirubin

For which of the following antibodies is the DAT most likely to be negative when testing a newborn for possible HDFN?

anti-A The DAT is most likely to be negative in ABO HDFN. It's possible that the washing done as part of the DAT may break the bonds between anti-A (or anti-B) and the newborn's poorly developed A (or B) antigens.

Lectin from the seeds of Dolichos biflorus is a source of which of the following?

anti-A1

When given during pregnancy, RhIg may cross the placenta and cause a positive DAT in the newborn.

True Because RhIg contains IgG anti-D, it can cross the placenta and sensitize fetal Rh positive red cells. Affected infants may be born with a weakly positive DAT, but significant hemolysis does not occur.

A rosette test to screen for FMH is contraindicated if the newborn is weak-D positive.

True The statement is true. A rosette test cannot be done to screen for FMH if the fetus is weak-D positive because false negatives may result. In this test, a sample of maternal blood is incubated with Rho(D) immune globulin, which will bind to fetal Rh positive cells. When a weak D antigen is expressed, the Rho(D) immune globulin may not bind entirely, which will not allow for rosetting to form once the indicator cells have been added.

If the mother is a Rh immune globulin (RhIg) candidate, blood safety standards mandate that a test for weak D is compulsory when initial D typing shows a newborn to be Rh negative.

True Weak D red cells can stimulate production of anti-D. Therefore, infants born to mothers who are RhIg candidates must be tested for weak D. RhIg is given to Rh negative women who deliver infants who are Rh positive or weak D.

Which of the following is true of the classic Bombay phenotype?

Two hh genes are inherited at the H locus

Which of the following contains all the possible phenotypes that could be the result of parents who are type O and type A:

Type A or type O only The group O parent can only supply an O gene to the offspring (OO) but the A parent could provide either an A or an O (AO). Therefore the only two possible combinations is OO or AO.

Which of the following signs and symptoms may be associated with immediate transfusion reaction, but is NOT usually associated with delayed hemolytic transfusion reaction?

Unexplained bleeding from surgical site Unexplained bleeding is associated with immediate hemolytic transfusion reactions, but is not usually associated with delayed hemolytic transfusion reactions. The bleeding results from disseminated intravascular coagulation (DIC) due to ABO antibodies causing intraventricular hemorrhage (IVH).

To detect the presence of blocking antibodies fixed on the red cells of a newborn infant:

Use the direct antiglobulin test

What is the purpose of a major crossmatch?

Verify donor ABO packed-cell compatibility

Which of the following are not appropriate indications for the use of fresh frozen plasma:

Volume expansion

An O pos patient has no reactions at immediate spin but both screen cells are positive and all antibody panel cells are reacting 1+ at AHG. The auto control is 1+. What would you suspect to be the cause?

Warm autoantibody

Which of the following is the most common type of autoimmune hemolytic anemia?

Warm autoimmune hemolytic anemia.

What is the basic technique employed to detect antibodies with a commercial set of panel cells?

Indirect antiglobulin test

What component is indicated for patients who receive directed donations from immediate family members to prevent transfusion-associated graft versus host disease (TA-GVHD)?

Irradiated Red Blood Cells

Which one of the following blood group systems may show a cell typing change during pregnancy?

Lewis

Which one of the following procedures is used for the proper preparation of Platelet concentrate from random whole-blood donors?

Light spin followed by a hard spin Feedback The first step in preparation of Platelets from random whole blood donors is a low-speed centrifugation (light spin). This allows the platelets to remain in the plasma portion of the collection container. The plasma (dubbed "platelet-rich plasma") is then centrifuged at a higher velocity (hard spin) that forces the platelets to the bottom of the satellite bag. The platelet-poor plasma is expressed into another container and the residual platelets that remain in the bag are resuspended in a small volume of plasma. Question Difficulty: Level 4

HLA-A and HLA-B antigens can be detected using which of the following techniques?

Lymphocyte cytotoxicity

Which of the following is responsible for causing graft-versus-host reactions:

Lymphocytes

What is the first line treatment for moderate to severe hemophilia A?

Lyophilized Factor VIII concentrate

When testing a patient with a warm autoantibody, which of the following is the most important concern?

Masking of other clinically significant antibodies Feedback Reactivity of a warm autoantibody may mask the presence of other clinically significant antibodies which adds to the possibility of a transfusion reaction. Further testing should be performed on the sample to determine if other antibodies are present. Question Difficulty: Level 3

In HDN which of the following antigen-antibody reactions is occurring:

Maternal antibody against fetal antigen

Which of the following set of conditions would NOT allow HDN to occur as a result of Rh incompatibility:

Mother Rh-negative, father Rh-negative

In immunohematology, an antithetical relationship exists between M antigen and which of these antigens?

N An antithetical relationship exists between the antigens M and N. Antithetical relationships occur in situations where for a given locus, only one of two genes may be inherited, the alleles are termed antithetical alleles.

Which of the following patients are at risk for transfusion-associated graft versus host disease (TA-GVHD) and require irradiated cellular blood products? (Choose all that apply)

Neonates less than 4 months of age Recipients of donor units known to be from a blood relative. Patient receiving chemotherapy who are immunocompromised.

What are the possible ABO genotypes of offspring of parents whose genotype is AO and BO:

OO AO BO AB

What are the possible ABO genotypes of offspring of parents whose genotype is AO and BO

OO AO BO AB The offspring from this set of parents can yield each blood type: OO, AO, BO, AB. This can be determined by the utilization of a simple punnett square. There is a 25% chance for inheritance of each blood type.

If an individual has blood type O, which of the following are possible genotypes?

OO only Feedback Individuals who type as group O must have two O genes present since both the A and B genes, if present, would have produced recognizable antigens. Question Difficulty: Level 3

Which of the following might cause a false positive indirect antiglobulin test:

Overcentrifugation

Which of the following blood groups is most frequently associated with cold agglutinins:

P Kell, Kidd, and Duffy are usually warm reacting IgG antibodies. The most common P system antibody is anti-P1, which is a frequently naturally occurring low titer IgM antibody, seen in most P2 individuals.

What is a common difference between paroxysmal cold hemoglobinuria (PCH) and cold hemagglutinin disease (CHD)?

PCH is caused by an IgG antibody while CHD is usually caused by an IgM antibody

The term used to describe patients with absence of Rh antigens is:

Rhnull Rhnull individuals have no Rh antigens. Rhmod individuals show reduced and varied reactivity with Rh antigens.

A patient with Multiple Myeloma has the following reactions in the ABO typing: Anti-A= w+ Anti-B = w+ Anti-A,B = w+ Auto control = w+ A1 Cells = 4+ B cells = 4+ What is probably causing these results?

Rouleaux

An urticarial transfusion reaction is characterized by:

Rash and hives

Rh antibodies generally:

React best at 37 degrees Celsius

IgM antibodies directed against red cells generally:

React best at room temperature

IgM antibodies produced against red blood cells generally:

React best at room temperature

Anti-H:

Reacts more strongly with group A2 cells than with group A1 cells A2 cells contain more H antigen than A1 cells.

Before testing all cord cells should be thoroughly washed in order to:

Remove Wharton's jelly The proteins found in Wharton's jelly can interfere with blood typing.

In the Coombs phase of a crossmatch, what is the proper procedure to follow if the Check Cells give a negative reaction?

Repeat procedure with new AHG reagent and check the cell washer

When performing an antibody screen, both the screen cells are 4+ at immediate spin and W+ at AHG. The antibody panel shows 4+ reactions at immediate spin and W+ reactions at AHG and there is no specific match to the reaction pattern. The auto control is negative. What would be a logical next step?

Repeat testing using warmed patient sample and reagents and just do AHG reading

When administering Fresh Frozen Plasma, which one of the following is considered standard blood bank practice?

Should be ABO compatible with the recipient's red blood cells

Fresh frozen plasma :

Should be transfused within 24 hours of thawing

A patient's serum reacts with all reagent red cell samples. The autocontrol is negative. An alloantibody to a high incidence antigen is suspected. Which of the following would be most likely to be a compatible donor:

Siblings

Enzyme panel reactions cannot be used as the only source of rule-outs during antibody identification. Why is this?

Some antigens are destroyed and may not detect an antibody that is present

Which of the following is the MOST IMPORTANT first step to take when a patient is transfused with un-crossmatched RBCs that turn out to be incompatible?

Stop any transfusion in progress.

What is the first step a transfusionist should take when a transfusion reaction is suspected?

Stop the transfusion, but keep the intravenous line open with saline.

Patients with antibody to the following antigen are immune to Hepatitis B:

Surface antigen

Transfusion related acute lung injury (TRALI) is a serious blood transfusion complication that can be characterized by:

Respiratory distress Non-cardiogenic pulmonary edema A mortality rate of 5-10% All of the above

The prozone effect can be described by all of the following EXCEPT:

Results in a false positive reaction

The use of cells with known blood groups to confirm ABO typing is known as:

Reverse grouping

Which of the following antibodies can be enhanced using enzyme panels during antibody identification in the blood bank?

Rh Lewis Kidd The Rh system antibodies along with Lewis and Kidd blood group antibodies can show an enhanced reaction when analyzed with enzyme treated panels. The antibodies which can be destroyed or weakened by enzyme treated panels are: the MNS group (though s can be variable), Duffy, and Xga.

If a pregnant female who was injected with RhIg antenatally has anti-D at delivery, routine antibody titration to determine the titer of the anti-D is considered a good practice.

False Routine antibody titration is not considered a good practice. If the antibody strength is weak (e.g., 2+ or less, consistent with RhIg administration), titer cannot differentiate between passive and immune anti-D.

The Kleihauer-Betke test is based on the principle that red cells containing fetal hemoglobin are more susceptible to acid elution than cells containing adult hemoglobin.

False The opposite is true. The Kleihauer-Betke test is based on the principle that red cells containing adult hemoglobin are more susceptible to acid elution than those containing fetal hemoglobin.

An anti-D titer of 4 in an Rh negative female who has received antenatal RhIg indicates that the anti-D is passive not immune.

False The statement is false. An anti-D titer of 4 is consistent with both passive and immune anti-D.

The seeds of the Dolichos biflorus plant will agglutinate A2 cells but not A1 cells.

False Anti-A1 lectin is extracted from the seeds of Dolichos biflorus. This reagent will agglutinate A1 cells, but not A2 cells.

Today, amniocentesis is preferred to Doppler sonography for monitoring the severity of hemolytic disease of the fetus and newborn (HDFN)

False Because Doppler sonography is noninvasive, it is a safer alternative to amniocentesis for fetal monitoring and has largely replaced serial amniocentesis for predicting severity of HDFN.

Today, amniocentesis is preferred to Doppler sonography for monitoring the severity of hemolytic disease of the fetus and newborn (HDFN).

False Because Doppler sonography is noninvasive, it is a safer alternative to amniocentesis for fetal monitoring and has largely replaced serial amniocentesis for predicting severity of HDFN.

Delayed hemolytic transfusion reactions (DHTR) typically occur 3 hours after transfusion.

False Delayed hemolytic transfusion reaction occur 3 - 7 days after transfusion. The reaction is caused by a secondary immune response which requires time for enough antibody to be produced by the patient to cause signs and symptom of extravascular hemolysis.

Hemoglobinuria and hemoglobinemia will be present in reactions where there is extravascular hemolysis.

False In extravascular hemolysis, there is no release of free hemoglobin, and therefore no hemoglobinemia or hemoglobinuria. Sensitized red cells are removed from circulation by the monocytes and macrophages in reticuloendothelial system.

RhIg prevents anti-D production mainly by clearing antibody-sensitized D-positive rbc from maternal circulation.

False Initially researchers thought that RhIg prevented anti-D production by clearing the infant's D-positive rbc sensitized with maternal anti-D, but we now know this is not true. Currently, the mechanism by which RhIg prevents immunization to the D antigen is poorly defined. However, research shows that it likely involves down-regulation of antigen-specific B cells, ie., the B cells do not differentiate into antibody-excreting plasma cells as they normally would when presented with foreign antigens.

Blood safety standards such as AABB Standards directly specify that an electronic crossmatch cannot be done when an Rh negative female has an anti-D consistent with antenatal RhIg administration.

False The statement is false. Current blood safety standards do not directly address whether an electronic crossmatch is contraindicated when anti-D consistent with antenatal RhIg administration is present. Laboratories are left to develop their own policies.

For infants born to Rh negative females, a test for weak D is optional when initial D typing shows the newborn to be Rh negative.

False The statement is false. For such infants, blood safety standards require a weak D test to be performed when initial D typing shows the newborn to be Rh negative.

When monitoring maternal antibody strength using a doubling dilution, an increase in titer from 16 to 32 is considered a significant rise in titer.

False The statement is false. To be significant, a rise in titer needs to be a two tube increase or more. A rise in titer from 16 to 32 in a doubling dilution is only a one tube difference.

The prozone effect ( when performing a screening titer) is most likely to result in:

False negative Prozone effect (due to antibody excess) will result in an initial false negative in spite of the large amount of antibody in the serum, followed by a positive result as the specimen is diluted.

The Rh nomenclature which uses the letters DCE is found in which of the following genetic models:

Fisher -Race Fisher and Race first proposed the existence of the three closely linked genes, and used the DCE terminology to describe their theory.

Which of the following methods is MOST reliable for determining the appropriate dosage of Rh immune globulin to give to an identified Rh immune globulin candidate after delivery?

Flow cytometry

Which of these methods is the MOST accurate and reliable for determining the volume of fetal whole blood or red blood cells in the maternal circulation in order to evaluate fetomaternal hemorrhage?

Flow cytometry

Match the correct components with their appropriate grouping:

Forward Grouping Patient Red Cells Reverse Grouping Reagent Red Cells Forward Grouping Reagent Antisera Reverse Grouping Patient Serum

Which of the following is the MOST likely discrepancy seen when a person demonstrates an "acquired B-like" phenomenon?

Forward typing appears to be AB, but reverse groups like A

Which blood component is the most commonly used component for the replacement of multiple coagulation factor deficiencies in bleeding patients?

Fresh Frozen Plasma

Which one of the following red blood cell phenotypes will NOT react with Anti-Fy3?

Fy(a--b-)

The term "allogenic blood" is synonymous in meaning with blood that is:

donated by a donor for a recipient other than the donor to use

Which D variant has a qualitative difference in the D antigen that allows individuals with the D variant to produce anti-D? Select all that apply.

partial D partial weak D

The BEST way to thaw Fresh Frozen Plasma is to thaw it by:

placing the FFP in a 37º C or lower in a water bath

Which type of blood component is most implicated in bacterial contamination?

platelets

A patient front-grouped as AB but back-grouped as O. The auto control was positive. If the testing was performed correctly, what is the most likely cause of this discrepancy?

presence of a cold autoantibody

Why is Rh immune globulin (RhIG) administered within 72 hours of delivery or miscarriage to an Rh-negative mother if the newborn is found to be D-positive or weak-D positive?

prevent fetal cells from initially sensitizing the mother

All of the following are reasons for conducting compatibility testing EXCEPT:

prevent recipient alloimmunization Compatibility testing is performed by the blood bank laboratory to detect serologic incompatibilities that might result in decreased survival of donor red cells in the transfused patient. The term compatibility testing, also known as pretransfusion testing, describes a set of procedures required before blood is issued as being compatible. Compatibility testing includes verification of ABO and Rh, selecting the proper blood products for transfusion, detecting antibodies against donor cells, and to avoiding transfusion reactions. It does not include the prevention of recipient alloimmunization since patients and donor units are not phenotyped for every antigen to prevent alloimmunization.

Based on the following results obtained against a patient's red cells, what will the genotype look like in this example? Anti-C = 4+ Anti-c = 4+ Anti-E = 0 Anti-e = 4+ Anti-D = 0

r'r Based on the positive antigen results given, the most likely Fisher-Race phenotype is Cde/cde. It is necessary to convert between Wiener system shorthand nomenclature to Fisher-Race nomenclature in order to make this determination. For example: Wiener Fisher-Race Ro Dce R1 DCe R2 DcE Rz DCE r dce r' dCe r" dcE ry dCE

Below is a listing of two commonly used terminology systems for genes and antigens. Use this information to answer this question: If a recipient has anti-c, which donor unit should be selected? Wiener Fisher-Race (haplotype) Ro Dce R1 DCe R2 DcE R DCE r dce r' dCe r" dcE ry dCE

r'ry It is necessary to convert Wiener system shorthand nomenclature to Fisher-Race nomenclature in order to make this determination. We can also combine the designations into genotypes. For example, r'ry would correspond to dCe/dCE. Therefore, this donor unit would not have the "little c" antigen(hr') and would be the correct answer. NOTE#1: blood bankers know that there is NO "little d" antigen, but they like to write it as a "place-holder for big D. NOTE #2: When referring specifically to an antigen: Rho = D; rh' = C; rh" = E; hr' = c and hr" = e.

The most frequent genotype among Rho (D) -negative persons is:

rr

What is the MOST common cause of acute hemolytic transfusion reactions?

Clerical errors

Match substance(s) secreted with their respective blood groups:

AB A, B, & H B B & H A A & H O H

What is the proper storage temperature for thawed Cryoprecipitate?

20 - 24 ºC.

IgM Antibodies: anti-M, anti-I IgG Antibodies: anti-K, anti-Jka

Anti-HIV-1

What is Coombs sera comprised of:

Anti-human globulins

Patients with diseases that require chronic transfusions are at risk for iron overload.

True

Which of the following is the proper storage temperature for fresh frozen plasma:

- 20 degrees Celsius FFP must be separated from RBCs and frozen solid at less than or equal to -18 degrees Celsius within 8 hours of collection.

The appropriate dosage of Rh immune globulin (RhIg) to administer post-delivery to an Rh-negative mother delivering an Rh-positive child is calculated based on the estimated volume of fetal bleed. What is the value of x in the formula given below that is used to calculate RhIg dosage? Number of vials of 300 µg RhIg = volume of fetal bleed/x mL Enter the number in the box below that is represented by x in the formula; do not spell out the number.(e.g., use "5" and not "five").

- 30 A 300 µg vial of RhIg can prevent immunization to a fetomaternal hemorrhage (FMH) of 30 mL of D-positive whole blood. Using the estimated volume of fetal bleed determined by the Kleihauer-Betke test or flow cytometry, the number of vials of RhIg (300 µg) to inject is calculated as follows: Number of vials of 300 µg RhIg = volume of fetal bleed/30 mL. In the interests of safety some American organizations recommend the following to deal with decimal points: If the number to the right of the decimal point is <5, round down and add 1 vial (e.g., 1.4 = 1 +1 = 2 vials) If the number to the right of the decimal point is greater than or equal to 5, round up and add 1 vial (e.g., 1.7 = 2 +1 = 3 vials).

Red Blood Cells, Frozen that have been prepared with high glycerol methods (40% glycerol) can be stored up to 10 years if held at which of the following temperatures?

- 65o C or lower

Match the blood types from the drop-down boxes with the appropriate descriptions to the right of the boxes.

- A(2) A antigens on RBCs and Anti-B antibodies in serum, which may also contain Anti A1 antibodies. - A(1)B A, A1 and B antigens on RBCs and no antibodies in serum. - B B antigens on RBCs and Anti-A antibodies in serum. - O No antigens on RBCs and Anti-A, Anti-B, and Anti-AB antibodies in serum. - A(2)B A and B antigens on RBCs and either Anti-A1 antibodies or no antibodies in serum. - A(1) A1 and A antigens on RBCs and Anti-B antibodies in serum. Type A2 has A antigens on the red blood cells with anti-B and possibly anti-A1 antibodies in the serum. Type A1B has A and B antigens on the red blood cells with neither anti-A nor anti-B in the serum. Type B has B antigens on the red blood cells and anti-A in the serum. Type O has no A or B antigens on the red blood cells, and anti-A, anti-B, and anti-AB antibodies in serum. Type A1 has A antigens on the red blood cells and anti-B in the serum.

All of the following are benefits of autologous donation except:

- Are always on hand in case of an unexpected emergency Autologous units must be drawn before they are needed, and must be readily available, therefore are generally not of use in emergencies. (Reduces exposure to infectious agents, Reduces demand for homologous blood, Eliminates sensitization to cellular blood components)

Which procedure used to obtain a fetal blood sample to monitor severity of HDFN can also be used to deliver intravenous transfusions?

- Cordocentesis Cordocentesis, also known as percutaneous umbilical blood sampling or PUBS can be used to obtain fetal blood samples for testing and to deliver intravenous transfusions.

Given the Wiener and Fisher-Race nomenclatures shown below, which offspring is NOT possible from a mother who is RoR1 and a father who is R1r? Wiener Fisher-Race (haplotype) R0 Dce R1 DCe R2 DcE Rz DCE r dce r' dCe r" dcE ry dCE

- DcE/DcE - DCe/DcE - DCe/dCe It is necessary to convert Wiener system shorthand nomenclature to Fisher-Race nomenclature in order to make this determination. The mother's Fisher-Race genotype for Wiener R0R1 would be Dce/DCe. The father's Fisher-Race genotype for Wiener R1r would be DCe/dce. The baby's Fisher-Race genotype for Wiener cannot be R2R2, (DcE/DcE) or R1R2 (DCe/DcE). Neither parent has the rh (E) antigen. The last choice is also not a possible offspring haplotype, as neither parent carried a dCe haplotype that could have been passed down.

The Kleihauer-Betke test is used to:

- Differentiate between maternal and fetal red cells The hemorrhage is estimated using the following formula: Fetal cells X maternal blood volume* / Total cells counted = Fetomaternal hemorrhage (mL of whole blood) * Maternal blood volume is estimated at 5000 mL

Which of the following tests are suitable for quantifying the size of fetomaternal hemorrhage (FMH)? Select all that apply.

- Flow cytometry - Kleihauer-Betke test Of these methods, only flow cytometry and the Kleihauer-Betke test can quantify the size of a FMH. The rosette test (but not the weak D test) is suitable to screen for FMH.

ABO antibodies are classified as which of the following two immunoglobulin classes?

- IgG - IgM

Which of the following is the predominant immunoglobulin class for anti-A and anti-B antibodies?

- IgM The predominant immunoglobulin class for the B antibodies produced by individuals with group A phenotype and the A antibodies produced by individuals with group B phenotype is IgM. IgG forms of anti-A and anti-B are also produced, but are not the predominant immunoglobulin class.

Can an autologous donor donate blood at 4 PM on Monday if she is having surgery at 10 AM on Wednesday?

- No, the minimal allowable time between the last donation and surgery is 72 hours. Although it is generally accepted that the minimal allowable time between the last autologous donation and surgery is 72 hours, it is highly unlikely that there will be an appreciable increase in red blood cell mass in this short of a time. Therefore, some facilities may have a cutoff time that is further out than 72 hours.

Which of the following types of packed RBCs could be transfused to a group O patient:

- None of the above Transfusion of red cells of any ABO type other than O to a group O patient is of course likely to cause a hemolytic transfusion reaction.

Which of the following is the most prevalent blood type found in the United States:

- O positive O positive is the most prevalent followed by A positive.

Which of the following is NOT required to be in a machine-readable format on a blood component label?

- Outdate The correct answer is D. The outdate may be handwritten. 21 CFR 606.121 requires this information be machine-readable: A unique collection facility identifier Lot number relating to the donor Product code ABO and Rh of the donor

Which of the following serological test results may occur in a patient experiencing a hemolytic transfusion reaction due to antibodies such as anti-c and anti-K? (Select all that apply.)

- Patient's antibody strength may initially decrease. - Patient may develop a positive DAT with mixed-field agglutination. - Patient's antibody becomes undetectable. - Patient's antibody increases in titer weeks later.

Which procedure should be followed when one cross-matched unit out of five is INCOMPATIBLE at the antiglobulin (AHG) phase?

- Perform a direct antiglobulin test (DAT) on the incompatible unit If the patient's serum is incompatible with only one unit at the AHG phase, the donor unit may have a positive DAT. If the test is positive, the donor unit should not be used for transfusion.

Which of the following actions should take place if a donor experiences numbness in the mouth during an apheresis procedure?

- Reduce the flow rate

A2B patients have or may have which of the following:

- Serum containing anti-A1. - Red cells that react with anti-A and anti-B. A2 and A2B individuals can produce anti-A1. In fact, about 25% of A2B individuals may have anti-A1 in their serum. Approximately 4% of all A2 individuals have naturally occurring anti-A1 in their serum.

What must be true for the antiglobulin phase of the serologic crossmatch to be omitted (i.e., immediate spin crossmatch is done)?

- The antibody screen must be negative. - There is no history of detection of unexpected antibodies

An acute hemolytic reaction may be caused by which of the following? (Choose all that apply)

- Transfusion of ABO incompatible blood. - Exposure to a red cell antigen when the corresponding antibody is present.

Which of the following Rh antigens is found the highest frequency in the Caucasian population:

- e The e antigen is present in 98% of the Caucasian population.

How would one prepare 3 mL of a 5% albumin working solution from a stock 30% albumin solution?

0.5 mL stock 30% albumin + 2.5 mL diluent Use the following formula to determine the amount of stock solution to use in preparing a working solution of lesser concentration: (% Stock solution) x (V1) = (% Working solution) x (V2) V1 = volume of the stock solution V2 = volume of the working solution Then: (30) x (V1) = (5) x (3 mL) 30 V1 = 15 mL V1 = 15 mL / 30 = 0.5 mL The total volume of working solution that is needed is 3 mL. Therefore, you would add 0.5 mL stock albumin to (3mL total volume working solution - 0.5 mL stock solution). In other words, add 0.5 mL stock 30% albumin to 2.5 mL diluent for a total volume of 3 mL of 5% albumin working solution.

The concentration of sodium chloride in an isotonic solution is :

0.85 %

The temperature range for maintaining red blood cell units (or whole blood units) during shipping is what?

1 - 10 ºC

A refrigerator used to store whole blood must be able to maintain a temperature in the ranges of:

1 - 6 degrees Celsius

If a potential donor has been previously transfused with blood products, he must be deferred from blood donation for:

1 year Feedback After an individual has been transfused with donor blood products, they must wait a minimum of 1 year before donating blood products. Question Difficulty: Level 6

Tube-based agglutination reactions in blood bank are graded from negative (0) to 4+. A reaction that has numerous small clumps in a cloudy, red background is:

1+ 1+ reaction has numerous small clumps and cloudy red supernatant 2+ has many medium-sized clumps and clear supernatant. 3+ has several large clumps and clear supernatant 4+ has one solid clump, no free cells, and clear supernatant

Once Fresh Frozen Plasma has been thawed at 30 - 37oC, it should be stored at what temperature?

1-6 ºC Thawed Fresh Frozen Plasma can be stored at 1 - 6oC for 1 to 5 days. It should be relabeled as "Thawed Plasma" and used for replacement therapy only in patients requiring stable clotting factors.

Which of the following candidates would be an acceptable blood donor? Blood pressure / Pulse / Temperature

110/72 66 37.2ºC (99.0 ºF) Systolic blood pressure should be no higher than 180, which eliminates candidate #2. Diastolic pressure should be no higher than 100, which eliminates candidate #5. Pulse should be between 50 and 100 beats/minute, which eliminates candidate #1. Temperature above 37.5oC (99.5oF) requires that the donor be temporarily rejected, which eliminates candidate #4. The only candidate that meets all of the requirements for blood donation is candidate #3.

A unit of Red Blood Cells that was collected on 15 June 2009 and frozen with glycerol at -80 degrees C on 21 June 2009 will expire on what date?

15 June 2019. Red Blood Cell units with the addition of a cryoprotectaive agent, such as glycerol, may be frozen for up to 10 years from day of collection. Frozen red blood cells must be stored at least -65 degrees C or colder.

How many international units (IU) of IgG anti-D are in 300 µg vial of RhIg?

1500 A 300 µg vial of RhIg contains 1500 IU of IgG anti-D. The conversion is one µg = 5 IU.

An Rh negative mother has just given birth to an Rh positive baby. Her physician suspects that she has experienced a fetal-maternal hemmorhage since her rosette test was positive. Upon performing the Kleihauer-Betke stain procedure, the percentage of fetal cells is found to be 0.85%. The mother's total blood volume is 4,565 mL. What dose of Rh Ig (RhoGam) should be administered to the mother?

2 vials Rh immune globulin, also known as Rh Ig or RhoGam, is used to help prevent an Rh negative mother from becoming sensitized to the D antigen from an Rh positive baby. To do this, vials of Rh Ig must be administered correctly. One full dose vial (300µg or equivalent) per 30 ml of D+ whole blood (15 ml D+ packed RBCs). To calculate how many vials are needed, the following formula can be employed: KB% x blood volume = volume of baby blood In this case: 0.85% (0.0085) x 4,565mL= 38.8 mL baby blood in maternal circulation 38.8mL / 30 mL per Rh Ig vial = 1.29 vials 1.29 vials can be rounded to 1 vial. In addition, an extra vial is always added. Therefore, 2 vials is the correct answer.

A pregnant female who was injected with RhIg antenatally has a positive antibody screen at delivery. The antibody has been confirmed as anti-D and reacts only in the indirect antiglobulin test. Many laboratories consider reaction strength to be an indicator of whether the anti-D is likely passive or immune. To be considered probably passive, an anti-D reaction strength is typically less than or equal to which reaction grade?

2+ Although passive anti-D can give varying reaction strengths depending on many variable, most laboratories consider 2+ or less to be consistent with passive anti-D from RhIg administration.

The proper storage requirements for granulocyte concentrates is:

20 - 24 degrees Celsius, < 24 hours

Platelets should be stored at what temperature:

20-24 degrees Celsius

After frozen RBC's have been thawed and washed they must be used within how many hours?

24

Granulocyte Concentrates MUST be administered within ______ of collection.

24 hours

Reconstituted deglycerolized Red Blood Cells that have been prepared in an open system must be used within _______.

24 hours

When a unit of packed RBC's is split using the open system, each half-unit must be issued within:

24 hours

Once the seal on a unit of packed red cells is broken, how long can the unit be stored refrigerated prior to administration:

24 hours If the seal on a unit of blood is broken, it must then be used within 24 hours and stored between 1 and 6 degrees C.

FFP that has been thawed at 30 - 37oC and maintained at 1 - 6oC must be transfused within ___________ after it has been thawed.

24 hours FFP that has been thawed at 30 - 37oC and maintained at 1 - 6oC must be transfused within 24 hours. In contrast, "Thawed Plasma" can be used for up to 5 days as a replacement therapy for patients requiring stable clotting factors. Keep in mind that these are two different component types and you are asked about FFP.

Which of the following donors could be accepted for blood donation?

24-year-old man who had a tooth extraction two months ago A tooth extraction that occurred two months ago would not be a reason for rejection as a blood donor. The remaining choices (22-year female who is currently pregnant; 40-year-old female who lived in London from 1988 - 1992; 44-year-old male who has been taking aspirin every four hours for the past two days) would all be reasons for rejection according to the current Uniform Donor History Questionnaire that was created by the FDA with input from the AABB donor history task force.

Antenatal RhIg (1500 IU dose) is typically given at how many weeks gestation?

28 A 1500 IU dose of RhIg is typically given at 28 weeks gestation.

What is the MAXIMUM interval during which a recipient sample may be used for crossmatching if the patient has recently been transfused or was pregnant within the past 3 months?

3 days

What is the maximum interval during which a recipient sample may be used for crossmatching if the patient has been recently transfused, has been pregnant within the past 3 months, or if relevant medical/transfusion history is unknown?

3 days

Antibody identification interpretations would be considered correct 95% of the time or have a P value of 0.05 if you have:

3 positive reactions to rule in an antibody and 3 negative reactions to rule out an antibody

Delayed hemolytic transfusion reactions (DHTR) usually occur within which time period?

3-10 days after transfusion

Rh immune globulin is manufactured to be capable of neutralizing _____________ milliliters of Rh positive whole blood.

30

A 300 µg dose of RhIg can suppress immunization to _____ mL of D-positive whole blood.

30 mL

A 300 µg dose of RhIg can suppress immunization to how many mL of D-positive whole blood?

30 mL One vial of 300µg RhIg can suppress immunization to a fetomaternal hemorrhage of approximately 30 mL of D+ whole blood

If the antigen frequencies for K = 0.09 and Fya = 0.66, what percent of type-specific units would be compatible for a patient with anti-K and anti-Fya?

31 The negative antigen frequencies are used in the formula below to determine the percent of type-specific units that would be compatible for the patient. Negative antigen frequencies are determined by subtracting the percent antigen frequency from 100%. The negative antigen frequency for K in this case is .91 (1.00 - .09) and the negative antigen frequency for Fya is .34 (1.00 - .66). % compatible units available = 100 x (Neg frequency #1 x Neg frequency #2...) For this case, the calculation is: % compatible units available = 100 x (0.91 x 0.34) = 30.94 or 31%

If an autologous blood donor weighs 35 kg, how much blood can be collected for later transfusion?

315 mL Volume to draw = (Donor's weight in kilograms / 50 kg (minimum weight requirement) x 450 mL (volume of unit to be donated) In this case, Volume to draw = (35 kg / 50 kg) x 450 mL = 315 mL

The shelf-life of whole blood collected in CPDA-1 is:

35 days

How long may blood be stored using CPDA-1 preservative prior to transfusion?

35 days Blood collected in CPDA-1 may be stored at 1 - 6 oC for up to 35 days.

Which of the following is the proper temperature to use when crossmatching in the presence of a cold antibody:

37 degrees Celsius Most antibodies that are inactive at 37 degrees Celsius, and active only below 37 degrees Celsius (i.e. cold reactive antibodies), are of little clinical significance.

Which of the following is the proper storage temperature for whole blood:

4 degrees Celsius Whole Blood should be stored between 1 - 6 degrees Celsius.

An Rh negative mother has just given birth to an Rh positive baby after 18 hours of strenuous labor. Her rosette test was positive. Upon performing the Kleihauer-Betke stain procedure, the percentage of fetal cells is found to be 1.9%. The mother's total blood volume is 5,000 mL. What dose of Rh Ig (RhoGam) should be administered to the mother?

4 vials Rh immune globulin, also known as Rh Ig or RhoGam, is used to help prevent an Rh negative mother from becoming sensitized to the D antigen from an Rh positive baby. To do this, vials of Rh Ig must be administered correctly. One full dose vial (300µg or equivalent) per 30 ml of D+ whole blood (15 ml D+ RBCs). To calculate how many vials are needed, the following formula can be employed: KB% x blood volume = volume of baby blood In this case: 1.9% x 5,000mL = 95 mL baby blood in maternal circulation 95mL / 30 mL per Rh Ig vial = 3.17 vials This equals 3 vials (after rounding), with the addition of 1 extra vial = the mother should have 4 vials of Rh Ig administered.

What percentage of glycerol is generally used when freezing red cells of rare phenotypes:

40 % Glycerol is a cryoprotectant that can be utilized in the process of storing rare phenotype cells, with a freezer storage temp of -80oC.. It is also beneficial for transfusion patients with multiple alloantibodies as these rare units can provide red cells that otherwise may not be available to treat these patients.

Which unit contains the proper number of platelets from a random donor?

5.8 x 10^10 Platelet concentrates are required to have a minimum of 5.5 x 10^10 platelets/unit. Random donor platelet concentrates typically contains between 5.5 and 8.5 X 1010 platelets suspended in about 50 mL of plasma. This is approximately 70% of the platelets which were present in the original unit of whole blood collected by the donor.

If an Rh group (DCe/dce) man marries an Rh group (dce/dce) woman, what is the probablitiy that their first child will be D-negative?

50% Since the inheritance pattern of the D-antigen is determined equally from the mother's and father's genotypes, the chances of producing a D-negative offspring is 50%. DCe (father) + dce (mother) = D positive dce (father) + dce (mother) = D negative

If parents have the blood group genotypes AA and BO, what is the possibility of having a child with a blood type of A?

50% The parents will each give one of their ABO genes, so the possibilities are as follows: AB, AO, AB, AO = 50% chance of A blood type, 50% chance of AB blood type

In order to prevent a loss of viability in platelet concentrates during storage the pH must be maintained above:

6.2

A patient transfused with two units of packed cells spiked a fever of 99.5oF and complained of chills five days after transfusion. The direct antiglobulin test (DAT) was positive with anti-IgG, but negative with anti-C3d. Compatibility testing was performed on the pre- and post-transfusion specimens. The post-transfusion specimen was incompatible with one of the donor units transfused. An antibody screen was done on both the pre- and post-transfusion specimens. An antibody was detected in the post-transfusion specimen only and identified by panel studies as anti-Jka. This transfusion reaction is most likely caused by:

A delayed hemolytic transfusion reaction Feedback Delayed hemolytic transfusion reactions (DHTR) are transfusion reactions that occurs 3 to 10 days after the transfusion. Usually, the blood appears serologically compatible at initial testing. Delayed reactions are common in patients who have been immunized to a foreign antigen from a previous transfusion or pregnancy. The antibody titers decrease over time so that the antibody was not detectable during pre-transfusion testing. Exposure to antigen causes an anamnestic response. Antibodies become detectable in the serum a few days after transfusion. Question Difficulty: Level 2

Which one of these physical exam results would cause a donor to be deferred?

A diastolic blood pressure of 110 mm Hg Diastolic pressure should be no higher than 100 mm Hg.

The most definite indication that a patient has been sensitized to a specific red cell antigen is:

A positive posttransfusion DAT in a patient with a previously negative DAT

Which of the following Fresh Frozen Plasma ABO types would be suitable for transfusion to an AB negative patient?

AB negative and AB positive only.

Which term listed below refers to the procedures that must be followed by a transfusion service when notification is received that a donor of a unit sent to the service now tests positive for an infectious disease?

A. Lookback

A laboratory employee who is performing an internal audit of routine ABO and Rh typing procedures notes that a technologist places two drops of Anti-D in a tube, centrifuges the tube for 20 seconds, and reads the reaction using a magnifying mirror. When questioned about the procedure, the technologist indicates that most staff use two drops of Anti-D reagent because the reactions are stronger with two drops. In addition to reviewing the facility procedure manual, what should be done to ensure regulatory compliance?

A. Review the anti-D manufacturer's current instructions for use.

Which of the following is the most common subgroup of A?

A1 The A1 subgroup is by far the most prevalent of all A subgroups, with A2 a distant second.

Which one of the following blood groups usually reacts LEAST strongly with anti-H?

A1 Of the blood groups that are listed, A1 has the least amount of H antigen and therefore would react least strongly with anti-H.

Which of the following blood groups reacts least strongly with Anti-H:

A1B The amount of H antigen present on red cells varies by blood group: O > A2 > B > A2B > A1 > A1B.

Which one of the following statements about anti-A1 is false:

A2 cells react with Dolichos biflorus lectin Dolichos biflorus lectin has anti-A1 specificity.

Red Cells Tested With Known Antisera Serum Tested With Known Red Cells Interpretation of ABO Group Anti-A Anti-B Anti-A,B A1 Cells B Cells 4+ 4+ 4+ 0 0 ?

AB

What are the possible ABO genotypes of offspring of parents whose genotype is AA and BB:

AB In this case, the only blood type inheritance possible is AB, as one parent (AA) has given the A gene, and the other parent (BB) is giving the B gene.

Which of the following is NOT a possible type for an offspring from the mating of an O and an AB (non-cis) individual?

AB The AB inheritance is not possible. In this case, the parents' genotypes are OO and AB, therefore one parent can only pass on the O gene.

Match appropriate genotype to its corresponding phenotype

AB - AB OO- O AA, AO- A BB, BO- B

Which of the following best describes the direct antiglobulin test principle:

AHG detects antibody already coated to red cells

If a child's phenotype is Group O, the phenotype of the mother of the child is Group A and the phenotype of the father is group B what are the possible genotypes of both parents?

AO and BO

Unexpected positive reactions encountered during forward ABO typing may be due to:

Acquired B antigen due to intestinal cancer

Which one of the following may cause a FALSE-NEGATIVE result with antiglobulin techniques?

Addition of AHG is delayed for 40 minutes or more after final saline wash

False negative results may occur with both the direct and indirect antiglobulin tests as a result of all of the following except:

Agglutination of red cells prior to addition of antiglobulin reagent

Which of the following refers to the most common procedure for donating whole blood for use by the general population:

Allogenic donation

A sample has reactions occurring at immediate spin and AHG in a panel that show varying reaction strengths. There is no obvious pattern that matches a particular panel cell or single antigen profile and the auto-control was negative. Which of the following is the most likely cause?

An IgM and an IgG antibody

Which listed transfusion reaction is MOST OFTEN associated with transfused patient's lacking IgA?

Anaphylaxis

A2 and A2B individuals can produce anti-A1. In fact, about 25% of A2B individuals may have anti-A1 in their serum. Approximately 4% of all A2 individuals have naturally occurring anti-A1 in their serum.

Anti-A Passive anti-D Antibody to a low frequency antigen All of the above are possible causes

The cause of the most severe life-threatening hemolytic transfusion reactions is:

Anti-A, Anti-B, Anti-A, B

Which of the following groups of antibodies generally reacts most strongly at 4o C:

Anti-A, Anti-P1 , Anti-Leb , Anti-M Anti-A, anti-P, anti-Leb, and anti-M all react best at 4o C as they are predominantly IgM antibodies. Other antibody group choices above include IgG antibodies such as anti-K, anti-s, anti-S, and anti-Fya, anti-Lub, etc. which react best at 37o C.

Based on the phenotype of the RBC screening cells, and patient results shown on the right, which of the following antibodies CANNOT be ruled out?

Anti-C Anti-Fya

A technologist performing an antibody identification in the blood bank has recorded the following results found in the worksheet below. Which antibody(ies) listed below cannot be ruled out?

Anti-C Anti-E Anti-K

The following results were obtained at delivery for a pregnant woman who received antenatal RhIg. ABO and Rh typing ABO Forward Group ABO Reverse Group Rh anti-A anti-B A1 cells B cells anti-D* 0 0 4+ 4+ 0 Antibody screen Cells Gel IAT* Screen cell l (R1R1) w+ Screen cell ll (R2R2) 1+ Screen cell lll (rr) 0 * IAT = indirect antiglobulin test Which of the following are possible causes of the positive antibody screen?

Anti-D (passive, from RhIg administration) Anti-D (immune) Antibody other than anti-D anti-D and another antibody All of the above are possible

If an R1 r patient received R2R2 blood, which of these antibodies could be produced :

Anti-E R1r = DCe/dce, R2R2 = DcE/DcE. Since E is not present in R1r anti-E could be produced.

Which of the following antibodies is detected primarily in the antiglobulin phase of the crossmatch:

Anti-Fya The Duffy system consists of Fya and Fyb. Antibodies to these antigens are IgG and are detected in the antiglobulin phase. Anti-M, Anti-B and Anti-P1 are typically IgM antibodies and may agglutinate saline suspended cells at room temperature.

What is present in the blood of an individual with the Bombay phenotype which will cause it to agglutinate with any non-Bombay individual's blood?

Anti-H Anti-H is produced by individuals with the Bombay phenotype because they are deficient in H, A, and B antigens. Due to the presence of anti-H in the serum of a person with the Bombay phenotype, only blood from another person with the Bombay phenotype may be transfused.

Group O blood cannot be transfused to a person with "Bombay blood" type because the Bombay individual has:

Anti-H Group O blood has the most H substance of all blood types; therfore, in a patient who has the Bombay phenotype (hh), the anti-H can react and cause incompatibility issues.

Why would a unit of group O blood never be administered to a Bombay patient:

Anti-H in recipient The classic Bombay phenotype (Oh) is characterized by the absence of A, B and H antigens, and the presence of anti-H, which will react from 4o to 37o C. Bombay patients must therefore be transfused only with blood from donors of the Bombay phenotype.

Which of the following antibodies will most likely not be detected on immediate spin?

Anti-Jka Jka and Jkb antibodies are nonagglutinating IgG antibodies, and are generally detected during the antiglobulin phase.

A technologist performing an antibody identification in the blood bank has recorded the following results found in the worksheet below. Which antibody(ies) are MOST LIKELY the cause of the agglutination reactions in this patient's sample?

Anti-Jka Anti-E The antibodies that are most likely present and causing the agglutination pattern observed are: Anti-E and Anti-Jka. Anti-Jkb, Anti-k, Anti-Leb, Anti-C, and Anti-c are each either ruled out and/or do not match the agglutination pattern present.

Which of the following antibodies is most often implicated as a cause of delayed hemolytic transfusion reaction (DHTR)?

Anti-Jka Of the antibodies that are listed, anti-Jka is most often reported as the cause of DHTR. Jkb is also well documented as a cause of DHTR. Antibodies that are also implicated in a DHTR include Kell, Rh, and Duffy system antibodies.

A technologist performing an antibody identification in the blood bank has recorded the following results found in the worksheet below. Which antibody(ies) are most likely the cause of the agglutination reactions in this patient's sample?

Anti-N Anti-E The antibodies that are most likely present and causing the agglutination pattern observed are: Anti-N and Anti-E. Anti-M, Anti-e, Anti-C, Anti-c, and Anti-S are either ruled out and/or do not match the agglutination pattern present.

Which Rh antibody might be produced if a unit of blood with Rh genotype DCe/dce is given to a patient with Rh genotype DCe/DCe?

Anti-c The c antigen is not present on the cells of the recipient in this case. Therefore, if a unit of blood which does in fact have the c antigen (DCe/dce as in this case) was to be transfused, there is a possibility of the recipient forming an antibody against the c antigen (anti-c).

The Direct Antiglobulin Test (DAT) is usually used to detect:

Antibodies coating red cells

The Indirect Antiglobulin Test (IAT) is usually used to detect:

Antibodies in the plasma

The two or three reagent cells used for antibody screening will detect which of the following:

Antibodies to common alloantigens

Which of the following statements is NOT true about the Lewis blood group:

Antigens are a structural component of the red cell membrane

Which of the conditions listed below must be met before a unit of Red Blood Cells that has been issued and returned can be reissued?

B. The container has a tamper-proof seal and the seal remains unbroken.

Which of the following group B antigens is generally associated with a mixed field reaction:

B3

What is the MOST common infective agent to be transmitted through blood transfusion that results in morbidity and mortality?

Bacteria

Which of the following best describes the primary function of antibodies:

Bind with antigen

The hh genotype gives rise to:

Bombay phenotype

Which is in the correct order from the lowest concentration of H antigen to the highest concentration of H antigen?

Bombay, A1B, A1, A2B, B, A2, O

Which of the procedures listed below will increase the platelet concentration in the preparation of platelets?

Centrifuge the blood at a low speed, remove the plasma and spin the plasma again at a high speed.

An aliquot of AS-1 red blood cells is being prepared from an intact packed cell unit using a sterile connection device. During the process of preparing an aliquot, the sterile device fails and blood drips onto the counter from the product tubing. What should be done with the primary unit?

Change the expiration date to 24 hours

Red blood cells with a positive DAT cannot be tested accurately with blood typing reagents that require an indirect antiglobulin technique unless they have been treated with ___________________ to dissociate IgG from the RBC membrane.

Chloroquine diphosphate Ficin ZZAP Any of the above

Which of the following statements is true regarding blood bank adverse event reporting to the FDA?

D. The initial notification to the FDA of a transfusion-related death must be made by fax, telephone, express mail, or electronically as soon as possible after the death is confirmed to be associated with the transfusion.

The use of the direct antiglobulin test is indicated in all the following except:

Detection of alloantibodies in serum

The most common Rh haplotype among whites is:

DCe Dce is found in 4% of whites and 44% of blacks. DCe is found in 42% of whites and 17% of blacks. DcE is found in 14% of whites and 11% of blacks. dce is found in 37% of whites and 26% of blacks.

What is the cause of neonatal alloimmune thrombocytopenia (NAIT)?

Destruction of a neonate's platelets by alloantibodies crossing the placenta from the mother.

What is the fundamental purpose of the full, pre-transfusion crossmatch?

Detect antibodies in recipient serum/plasma that react with donor red cells.

Essential components of compatibility testing include all of the following except :

Direct Antiglobulin Test (DAT)

When AHG or Coombs serum is used to demonstrate that red cells are antibody coated in vivo, the procedure is termed:

Direct technique DAT ( Direct Antiglobulin Test ).

What action should be taken if a large clot is noticed in a red blood cell unit while the product is being prepared for release to the patient?

Do not issue the product.

Which of the following is the most common reagent source for Anti-A1 (a reagent that is used to differentiate subgroup A1 from subgroup A2)?

Dolichos biflorous seeds

The major crossmatch is performed using:

Donor's red cells and recipient's serum or plasma

Based on the reactions below, indicate the correct blood group for each patient: Patient #1 Forward (Cell) Typing Reverse (Serum) Typing Anti-A Anti-B Anti-A,B A1 Cells B Cells 4+ 4+ 4+ 0 0 Patient #2 Forward (Cell) Typing Reverse (Serum) Typing Anti-A Anti-B Anti-A,B A1 Cells B Cells 0 0 0 4+ 4+ Patient #3 Forward (Cell) Typing Reverse (Serum) Typing Anti-A Anti-B Anti-A,B A1 Cells B Cells 4+ 0 4+ 0 4+ Patient #4 Forward (Cell) Typing Reverse (Serum) Typing Anti-A Anti-B Anti-A,B A1 Cells B Cells 0 4+ 4+ 4+ 0

Group AB Patient #1 Group O Patient #2 Group A Patient #3 Group B Patient #4

Which blood group is considered the "universal" donor when transfusing Red Blood Cells?

Group O Group O blood lacks A and B antigens so that it may be transfused to all other ABO groups in an emergency situation. Group O recipients, however can only receive group O cells.

Which of the following types of whole blood would be the least satisfactory to transfuse to a type AB patient:

Group O Group O whole blood contains both anti-A and anti-B which could react with the recipient's RBCs.

Which of the following red blood cells contain the most H antigen:

Group O cells

In an extreme emergency , if the ABO and Rh type are unknown which of the following should be given to the patient?

Group O, Rh negative blood "Universal donor", (a misnomer) is usually applied to group O, Rh negative blood. Although it may be necessary to use group O, Rh negative blood in an extreme emergency, it is preferable to use type specific blood for emergencies.

The antigen marker most closely associated with transmission of HBV infections is:

HBeAg

Which is the first marker (antigen or antibody) which will become positive after exposure to Hepatitis B:

HBsAg Feedback HBsAg is the first hepatitis antigen to appear after infection. The first antibody produced after infection with HBV is anti-Hbc; used to detectpresent or past infection. Anti-Hbe is present in individuals who have recovered from acute hepatitis B infection. Anti-HBs becomes positive later, during the recovery period. Question Difficulty: Level 3

Which of the following tests has been recommended by the FDA to replace the HIV-1 p24 antigen test in the screening of donated blood for infectious diseases?

HIV-1 NAT The FDA recommends that HIV-1 nucleic acid testing (NAT) be done in place of HIV-1 p24. Anti-HIV-1 and anti-HIV-2 are still required screening tests. HCV NAT is recommended as part of the battery of tests, but is not a replacement for HIV-1 p24 testing.

The most recent algorithm for HIV confirmatory testing in patients who are positive by ELISA is the:

HIV-1/HIV-2 antibody differentiation immunoassay

Which of the following patients represents an acceptable donor.

Hct is 41, BP is 110/80, Temp is 99.4, Pulse is 65, Age is 65, and Sex is male With certain exceptions, blood donors must be at least 17, must have a hematocrit of at least 38%, must have a blood pressure no greater than 180 systolic, and no greater than 100 diastolic. Their pulse must be regular and between 50 and 100 beats per minute. Their temperature must not exceed 99.5o F (37.5o C). Exceptions to these criteria should be evaluated by the blood bank physician on a case by case basis.

Which of the following is the most probable explanation for the results below? The following results were obtained: ABO/Rh Type DAT Ab. Screen and ID Mother A negative NT Positive anti-D Infant O positive 4+ IgG NT NT = not tested

Hemolytic disease of the newborn due to anti-D

Which of the following statements about high-frequency antigens is correct?

High-frequency antigens are common, but it's difficult to identify their corresponding antibodies

Which of the following consequences of severe hemolytic disease of the fetus and newborn (HDFN) is most associated with neonatal death before or shortly after birth?

Hydrops fetalis

When should a prewarm technique be performed?

If you suspect a cold antibody Prewarming of a sample with a suspected cold antibody will prevent the activation and binding of the antibody at room temperature. Warm autoantibodies are not affected by prewarming. IgG antibodies react at 37 ° C since that is the optimum reaction temperature.

A secondary immune response is generally associated with which of the following antibodies:

IgG

Which immunoglobulin class is able to cross the placenta from the mother to the fetus?

IgG Feedback Immunoglobulins of the IgG class are able to cross the placenta since they circulate as monomers; thus they are small in size. The other types of immunoglobulins are too large to cross the placental barrier, for example, IgM is a pentamer. Question Difficulty: Level 1

Which of the following antibodies is predominantly associated with the secondary antibody response:

IgG IgG - Secondary immune response IgM - Primary immune response

Which class of immunoglobulins is implicated in hemolytic disease of the fetus and newborn (HDFN)?

IgG IgG antibodies are implicated in HDFN. They are small enough to cross the placental barrier to enter fetal circulation.

Match antibody group with corresponding descriptor:

IgG - IgM Fixes complement via classical pathway IgM Largest antibody, pentamer IgG Crosses placental barrier IgE Immediate hypersensitivity

Polyspecific antihuman globulin (AHG) reagent used in antiglobulin testing should react with which one of the following?

IgG and C3d Polyspecific antihuman globulin (AHG) reagent contains both anti-IgG and anti-C3d.

Most blood group antibodies belong to which immunoglobulin classes?

IgG and IgM

Coombs control Check Cells used to verify negative reactions at the anti-globulin (IAT) phase are usually coated with:

IgG antibodies A control that is red blood cells coated with IgG antibodies is applied to each antiglobulin test interpreted as negative in order to identify a false-negative result.

Which of the following immunoglobulins is capable of fixing complement by the classical pathway:

IgG, IgM IgG1, IgG3, and IgM fix complement via the classical pathway. IgA fixes complement through the alternative pathway. Only IgG crossed the placental barrier.

FALSE-NEGATIVE results at the indirect antiglobulin phase of an antibody screening test using a tube method (i.e., not a Gel-method) are usually caused by which of the following?

Inadequate washing of the red cells

For transfusion services in the United States, which of the following incidents must be reported to the Food and Drug Administration (FDA) because of a biological product deviation?

Incident C: The wrong specimen was used to crossmatch a unit and the unit was issued.

Red Cells Tested With Known Antisera Serum Tested With Known Red Cells Interpretation of ABO Group Anti-A Anti-B Anti-A,B A1 Cells B Cells 4+ 4+ 4+ 1+ 0 ? Using the information provided above, select the correct ABO group.

Inconclusive; could be A2B with anti-A1 in the serum; further testing is necessary.

Red Cells Tested With Known Antisera Anti-A Anti-B Anti-A,B 4+ 4+ 4+ Serum Tested With Known Red Cells A1 Cells B Cells 1+ 0 Interpretation of ABO Group ? Using the information provided above, select the correct ABO group.

Inconclusive; could be A2B with anti-A1 in the serum; further testing is necessary. Feedback The forward type in this case suggests type AB; however, in the reverse type there is an unexpected presence of the anti-A1 antibody. This means that more testing must be performed to identify why this antibody is present. One possible solution is that the patient has an A antigen subtype. ABO reactions with reagent antisera and red cells are generally strong (3 - 4+); weak reactions (less than 3+) should be investigated. Question Difficulty: Level 2

What should be the first step performed to resolve a case where all forward and reverse ABO typing results are negative on a patient?

Incubate all testing tubes at 22ºc

Which symptom of HDFN is associated with low levels of glucuronyl transferase?

Jaundice

A delayed hemolytic transfusion reaction is most likely to be the result of which of the following antibodies:

Jka Jka (Kidd) antibodies are very dangerous; they disappear from circulation quickly and can cause severe delayed transfusion reactions.

Which of the following blood group antigen-antibody reactions is enhanced by using enzymes:

Jka Duffy antigens (Fya and Fyb) and M,N,S antigens are denatured by enzymes. Kidd antigens (Jka, Jkb) are enhanced by enzymes.

The McLeod phenotype is associated with which of the following antigen systems?

Kell

Which of the following antigens are well developed on fetal cells?

Kell

Which symptom of HDFN does phototherapy help prevent?

Kernicterus

Proteolytic enzyme techniques may be useful in identifying which of the following antigen groups:

Kidd system Antibodies to Kidd antigens are difficult to detect because they show dosage and therefore may not react with heterozygous cells, quickly fall to nondetectable levels in vivo, and sometimes require complement to react in vitro, thus losing reactivity during storage of serum. Reactivity of Kidd antibodies is enhanced by the use of enzymes.

A delayed hemolytic reaction occurring a week later is MOST likely caused by:

Kidd system antibodies

Delayed hemolytic transfusion reactions are usually caused by antibodies directed against what blood group system?

Kidd. The most likely causes of delayed hemolytic reactions are Kidd system antibodies. Both jka and jkb are often responsible for delayed hemolytic transfusion reactions.

ABO blood groups were discovered by:

Landsteiner Landsteiner first identified the presence of the separate red cell antigens A and B in the early 1900's.

Which one of these Lewis blood group system phenotypes usually produces anti-Lea?

Le(a--b-) This antibody is found in the serum of Le(a-b-) secretors.

Which Lewis antigen(s) will be exhibited on the red cells of adults who have the Le, Se, and H genes?

Leb

Which one of the following blood group antigens is not expressed, or only weakly expressed on cord blood cells?

Leb Newborns are phenotypically Le(a-b-).

Which of the following would NOT be included in a hemolytic transfusion reaction investigation?

Leukocyte antigen studies Leukocyte antigen studies would NOT be included in a hemolytic transfusion reaction investigation. The remaining procedures must be performed as part of the initial reaction investigation. A post-transfusion specimen should be sent to the laboratory for work-up. A clerical check should be performed to investigate possible errors in specimen labeling, blood product issuance, or patient identification. The plasma must be examined for hemolysis. A direct antiglobulin test (DAT) must be performed. The patient's ABO, Rh, and antibody screen should be repeated and confirmed. The blood product ABO/Rh can also be confirmed.

A patient with two or more documented febrile nonhemolytic transfusion reactions (FNHTRs) should receive __________ blood components.

Leukoreduced

Which of the following is generally considered equivalent to CMV seronegative RBC for use in an exchange transfusion to a newborn?

Leukoreduced RBC

Red Cells Tested With Known Antisera Serum Tested With Known Red Cells Interpretation of ABO Group Anti-A Anti-B Anti-A,B A1 Cells B Cells 0 0 0 4+ 4+ ?

O

Which one of the ABO groups listed below has the MOST H-antigen?

O The amount of H antigen varies depending on the ABO group that is inherited. Group O has the greatest amount of H antigen, followed by A2 then B, and then A1. A1B has the least amount of H antigen.

Match the blood groups on the left with corresponding results of forward typing on the right.

O Anti-A = negative and Anti-B = negative A Anti-A = positive (4+) and Anti-B = negative B Anti-A = negative and Anti-B = positive (4+) AB Anti-A = positive (4+) and Anti-B = positive (4+) Feedback Since the forward type is testing the patient cells against reagent antisera, the reactions noted as positive for the antibody contained in the reagent indicate the presence of these antigens on the surface of the patient red cells. Therefore, persons who are type A have A-antigen on their red cells, but no B antigen and a forward type will produce a positive reaction with anti-A reagent and a negative reaction with anti-B reagent. Patients who are type B have B-antigen on their red cells but no A antigen, resulting in a positive reaction with anti-B reagent and a negative reaction with anti-A reagent. Type AB has both A and B antigen and will therefore produce positive reactions for both A and B in a forward typing. Type O has neither A nor B antigens on the red cells and will produce negative reactions with both anti-A and anti-B. Question Difficulty: Level 3

Use the drop-down boxes to match the blood types (phenotypes) that will be expressed with the genotypes listed to the right of the boxes.

O OO A OA AB AB B BB A AA B OB

Which of the following packed red blood cell ABO types would be appropriate for transfusion to an O- negative patient?

O negative only.

In order to detect the presence of antibodies fixed on the RBC's of newborns one should:

Perform a direct antiglobulin test (DAT) on the cord blood Feedback A positive DAT is consistent with a diagnosis of hemolytic disease of the newborn. A hemoglobin performed on the cord blood serves as an indicator of the severity of the disease. Question Difficulty: Level 2

What should be done FIRST if a donor unit is found to be incompatible at the antiglobulin phase with several different recipients?

Perform a direct antiglobulin test (DAT) on the donor unit

A patient experiences a mild allergic reaction to a transfusion, including urticaria, erythema (skin redness), and itching. What is the most likely source of the allergen?

Plasma proteins Mild allergic reactions result from a patient's hypersensitivity to soluble allergens in the plasma of the donor unit. The blood recipient forms antibodies to these allergens that are bound to IgE on mast cells and cause the release of histamines. Allergen substances may be drugs or food consumed by the blood donor.

Which one of the following blood components would be MOST appropriate for a 9-yr old girl, with a low hemoglobin and low platelet count, who has bleeding gums?

Platelet Concentrate

Bacterial contamination is MOST likely in which of the following blood products?

Platelets

In blood bank agglutination reactions, the zeta potential (a force exerted by ions in the saline solution that causes repulsion between red blood cells in the saline suspension) can be reduced by treating the sensitized cells with:

Polyethylene glycol (PEG), albumin, or proteolyic enzymes Feedback Antigen-antibody reactions will not occur, or will be weak, if a force exerted by ions in the saline solution cause repulsion between two adjacent red blood cells. The zeta potential can be reduced by treating the red blood cells with proteolytic enzymes such as papain and ficin, or using various colloidal diluents such as albumin or PEG. Question Difficulty: Level 2

A 40-year-old female receives two units of Red Blood Cells during a surgical procedure. The patient has no prior history of transfusions. Seven days later, she presents with extensive bruising of the extremities and bleeding of the gums, with no additional symptoms. Her platelet count is 5 x 109/L ( reference interval 150 - 400 x 109/L). What is the most likely diagnosis?

Post transfusion purpura (PTP)

Gamma irradiation of cellular blood components is required in which of the following situations:

Prevent Graft-Versus-Host (GVH) disease

A solution of gamma globulins containing anti-Rh (D) is given to an Rh (D) negative mother to:

Prevent fetal cells from initially sensitizing the mother

What should be done with a box of 10 Red Blood Cell units that was received with a measured temperature inside the box at 15ºC.

Quarantine and destroy the products.

What is his MOST likely phenotype for a white male who has the following Rh antigens C,c,D,E,e?

R1R2 R1 = DCe, R2 = DcE Cannot be R1r, R0r, r'r'', or R1r' since the male has the D and the E antigen. The R1r, R0r, and R1r' phenotypes can be eliminated because the E antigen is missing in these phenotypes while the male actually carries the E antigen. The phenotype r'r'' can be eliminated since the male does indeed have a D antigen while in the r'r'' phenotype it is missing.

Given the following commonly used nomenclature systems, which one of the Rh genotypes listed below is heterozygous for the C antigen? WienerFisher-Race (haplotype) RoDce R1 DCe R2DcE RzDCE rdce r'dCe r"dcE rydCE

R1r R1r is the only genotype listed that is heterozygous for the C antigen. Weiner Fisher-Race R1r DCe/dce R1R1 DCe/DCe rr dce/dce R1R2 DCe/DcE R2r DcE/dce R2R2 DcE/DcE r'r dCe/dce r'r' dCe/dCe r"r dcE/dce r"r" dcE/dcE R0r Dce/dce

A mother's serologic results are shown above. Her newborn types as group A Rh positive with a (1+) positive direct antiglobulin test (DAT). Which of the following investigative tests would be most useful to resolve the cause of the positive DAT and should be done FIRST?

Test newborn's plasma against group A1 red cells and group O antibody screen cells by IAT.

In what way are the ABO serum antibodies unique among blood group systems?

The antibodies are naturally occurring.

Why do so few patients transfused with un-crossmatched red cells in an emergency experience a hemolytic transfusion reaction? Select the one best reason.

The incidence of unexpected red cell antibodies is relatively low.

Why is it dangerous to transfuse a person with type O blood with a unit of A blood?

The patient's anti-A would destroy the donor's cells with severe consequences to the patient.

Avidity is best described by which of the following statements:

The strength with which multivalent antigens and antibodies bind

Which one of the following statements about directed donations is true:

They are often perceived by the recipient or his family as safer than random donor units Feedback Directed donations can sometimes be perceived as safer than non-directed units; however, this is not the case. On occasion the donors for directed units may not be completely forthcoming with information about their health status, which could potentially harm the recipient of the blood product unit. Therefore, directed donor units are not necessarily safer than random donor units. Question Difficulty: Level 2

In preparing red cells for any elution method , one must be particularly careful to:

Thoroughly wash sensitized red cells Otherwise, serum antibody may contaminate the eluate.

In the interest of safety, it is the policy at XYZ hospital to always add one extra 300 µg vial of RhIG, regardless if the dosage calculation is rounded up or rounded down. After performing a Kleihauer-Betke test, a technologist in the laboratory at XYZ hospital calculates the fetomaternal hemorrhage to be 45 mL of fetal whole blood. How many 300 µg vials of RhIG should be administered to this woman?

Three The calculated bleed was 45 mL of fetal whole blood, the dosage calculations would yield 1.5 vials (45 mL fetal whole blood / 30 mL) = 1.5. One vial of Rh immunoglobulin (RhIG) is used for every 30 mL of fetal whole blood (or 15mL of fetal packed cells). This number, 1.5, would be rounded to 2 and an additional vial added. Thus, the final dosage would be 3 vials (900 µg or 4500 IU). In the interests of safety, if the number to the right of the decimal point is <5, round down and add 1 vial (e.g., 1.4 = 1 +1 = 2 vials) If the number to the right of the decimal point is greater than or equal to 5, round up and add 1 vial (e.g., 1.5 = 2 +1 = 3 vials).

For which of these reasons would a molecular method be used to determine a pregnant woman's Rh type?

To differentiate between weak D or partial D

An exchange transfusion has been ordered for a low birth weight infant. The physician has ordered irradiated Red Blood Cells for this purpose. Why is it necessary to irradiate the blood products for this patient?

To prevent transfusion-associated graft-versus-host disease (TA-GVHD)

Which one of the following types of transfusion reactions caused the highest number of transfusion-related fatalities that were reported to the U.S. Food and Drug Administration (FDA) from 2005 - 2009?

Transfusion-related acute lung injury (TRALI) Feedback In combined fiscal years 2005 through 2009, transfusion-related acute lung injury (TRALI) caused the higest number of reported fatalities (48%), followed by hemolytic transfusion reactions (26%) due to non-ABO (16%) and ABO (10%) incompatibilities. Complications of microbial infection, transfusion-associated circulatory overload (TACO), and anaphylactic reactions each accounted for a smaller number of reported fatalities. Reference: U.S. Food and Drug Administration Website. Fatalities reported to FDA following blood collection and transfusion: Annual summary for fiscal year 2009. Available at: http://www.fda.gov/BiologicsBloodVaccines/SafetyAvailability/ReportaProblem/TransfusionDonationFatalities/ucm204763.htm. Accessed Janary 7, 2011.

Kernicterus due to high levels of unconjugated bilirubin can cause brain damage in newborns suffering from severe HDFN.

True

Not performing direct antiglobulin tests (DATs) on newborns born to group O Rh positive mothers is acceptable good practice, providing there is appropriate surveillance and follow-up to detect hyperbilirubinemia.

True

Premedication with antihistamines may prevent an allergic reaction in patients with a history of multiple urticarial transfusion reactions.

True

RhIg prophylaxis is typically given antenatally to Rh negative pregnant females without knowing the Rh of the fetus.

True

The Kleihauer-Betke test used to quantitate FMH has poor reproducibility.

True

The ability to produce anti-D varies significantly among Rh negative individuals.

True

True or False: When antigen typing, mixed-field agglutination (MFA) due to red cell transfusion indicates RECENT transfusion, specifically transfusion within the past three months.

True

Tiny volumes of D+ red cells (e.g., as little as 0.1 mL) can stimulate production of anti-D in some individuals.

True Although the D antigen is very immunogenic, the ability to produce anti-D varies greatly among individuals. Some people will make anti-D after exposure to exceedingly small volumes of Rh positive red cells such as 0.1 mL

The serum of some group A individuals may agglutinate group A1 cells.

True Approximately 4% of individuals with A2 blood type will have Anti-A1 antibodies in their serum. Individuals with some rarer A subgroups may also have Anti-A1 in their serum. Agglutination will occur if the serum from any of these individuals is mixed with A1 red cells.

A patient with a positive DAT due to IgG cannot be reliably antigen-phenotyped using antisera that reacts by the indirect antiglobulin test (IAT).

True A positive DAT precludes antigen phenotyping by IAT. IgG-sensitized red cells will react with the antiglobuin serum whether the cells are antigen positive or negative.

A patient who received an autologous transfusion one week ago can be reliably antigen typed.

True Autologous transfusion does not preclude valid antigen typing. Problems occur when two red cell populations that are genetically different cause mixed-field agglutination.

Evaluating B-type natriuretic peptide (BNP) levels can be helpful in distinguishing transfusion-related acute lung injuries (TRALI) from cardiopulmonary edemas.

True Feedback It is important to rule out cardiac causes of pulmonary edema. One way of differentiating is evaluating the BNP level, which is known to be elevated in congestive heart failure and not TRALI. Question Difficulty: Level 3

At 28 weeks gestation, a group A Rh-negative female had a negative antibody screen and was injected with Rh immune globulin (RhIg). Anti-D is detected in the mother's serum at delivery. A panel is performed that excludes the presence of other antibodies. The most likely conclusion would be passive anti-D due to RhIg administration.

True If a mini-panel or full-panel is performed and excludes other antibodies, report "probable passive anti-D due to RhIg administration;" "passive anti-D consistent with recent RhIg administration" or similar.

For those facilities that in the interest of safety use a special calculation for RhIg dosage, regardless if they round up or round down, they always add one vial.

True Regardless of rounding up or down, when calculating RhIg dosage, such facilities always add one vial. For example: 1.4 = 1 +1 = 2 vials 1.5 = 2 +1 = 3 vials

Ectopic pregnancy is an indication for administering RhIg to an Rh negative woman.

True RhIg is indicated in any circumstance in which D-positive fetal red cells may enter the circulation of an Rh-negative female, including ectopic pregnancy.

The incidence of HDFN due to anti-D varies significantly according to race and ethnicity.

True The incidence of HDFN due to anti-D varies significantly according to race and ethnicity because of the frequency of D in different populations. HDFN is rare in a population that is almost exclusively Rh-positive.

Which of the following antibodies usually show enhanced agglutination with the use of proteolytic enzymes?

anti-Jka, Jkb, C, and E Enzyme techniques are particularly useful in the identification of antibodies in the Rh system (e.g., anti-C and E) and in the Kidd system (e.g., anti-Jka and Jkb). Enzymes destroy some antigens, such as M, N, S, Fya, and Fyb. Therefore the corresponding antibodies would not be detected.

For the test results shown above, which of the following antibodies is most likely to be causing the newborn's positive DAT?

anti-K The correct answer is anti-K. Both the mother and infant are group A, eliminating ABO HDFN. We are given the cell ID information for each of the screening cells. Screen cell 1 is R1R1. Therefore these antigens are present: D,C, e. Screen cell 2 is R2R2. Therefore these antigens are present: D,E,c Screen cell 3 is rr. Therefore these antigens are present: c,e The only cell that reacted was Screen cell 3. Both c and e are also present on the nonreactive cells. The only choice that has not been eliminated is anti-K. Also, the antibody screen reaction pattern (only cell 3 positive) is typical of anti-K.

Of the following blood group antibodies, which has been most frequently associated with severe cases of hemolytic disease of the fetus and newborn (HDFN)?

anti-K Of those listed, anti-K is most frequently associated with severe forms of HDFN. Anti-A,B is frequently implicated in HDFN, but the disease is generally mild, often subclinical. Anti-Lea is not implicated in HDFN for two reasons; the antibody is generally IgM and the Lewis system antigens are poorly developed at birth.

What is the most prudent step to follow to select units for cross-match after recipient antibodies have been IDENTIFIED?

antigen type patient cells and any donor cells to be cross-matched

Post-transfusion purpura (PTP) is characterized by which of the following?

appearance of purpura platelet count of less than 10,000/uL mucousal membrane bleeding

In performing an AHG test it is important to completely wash the red cells in order to:

avoid neutralization of the anti-human globulin serum Inadequate cell washing will lead to unbound antibody remaining in the red cell suspension. This residual unbound antibody would be available to neutralize the AHG (Coombs serum) so it will not react with red cells bound with antibody.

What is the explanation for a discrepancy in the following front type and back type in a newborn? Anti-A = neg Anti-B = neg Anti-A,B = neg Al cells = neg B cells = neg O cells = neg Auto-control = neg

baby's antibodies are undeveloped At birth, ABO antibodies are generall absent. ABO antibody production begins at about 3 - 6 months of age.

During routine inspection, a unit of unexpired blood was noticed to have a black color with numerous small clots. What is the likely cause for this observation?

bacterial contamination

What is the FIRST thing blood bank staff should do when investigating a transfusion reaction?

check for clerical errors in all steps of the pre- and post processes

Which one of the following blood components contains the MOST factor VIII concentration per ml?

cryoprecipitate

Which of the following is not a major Rh antigen:

d Feedback There is no known d antigen. Question Difficulty: Level 3

What other component(s) can be shipped together with Fresh Frozen Plasma (FFP)?

frozen RBC's and cryoprecipitate

What should be done if all forward and reverse ABO results are NEGATIVE?

incubate at room temperature or 4°C to enhance weak expression

Most antibodies present in cord blood are of ________ origin.

maternal

A positive Coombs control test (check cells) in a cross match BEST indicates that the:

negative antiglobulin test was actually negative

Antibodies are ruled out using panel cells that are homozygous for the corresponding antigen because:

stronger reactions seen with homozygous cells

What is the MOST likely explanation for the following results? Anti-A = 1+ Anti-B = negative A1 Cells = 1+ B Cells = 4+

subgroup of A with anti-A1 This is an ABO discrepancy in which the forward ABO group appears as a group A, but the reverse grouping appears as a group O.

When a group O patient's serum is not compatible with O cells, and the auto control is negative, what is the MOST likely cause?

the patient is a Bombay phenotype

Which one of these physical examination results may be cause for rejecting a blood donor?

weight of 100 pounds (45.4 kg) The AABB standards require that the maximum whole blood volume that can be removed during a blood donation is 10.5mL/kg of donor weight (including samples removed for testing as well as blood unit volume). In general, a donor weight of 110 pounds or greater would meet this standard. The maximum whole blood volume that is removed by a blood donation (450 mL or 500 mL) would be exceeded if a donor only weighed 100 pounds.


Conjuntos de estudio relacionados

Stuff you got wrong accounting ch. 10

View Set

Human Development Chapters 11-14

View Set

Discrete Probability Distributions

View Set

How to Read a Book: The Classic Guide to Intelligent Reading by Mortimer J. Adler and Charles Van Doren

View Set

CFA Level II Quick Sheet - Quantitative Methods

View Set

Chapter 14: Dividends and Dividend Policy

View Set